User avatar
 
ManhattanPrepLSAT1
Thanks Received: 1909
Atticus Finch
Atticus Finch
 
Posts: 2851
Joined: October 07th, 2009
 
This post thanked 3 times.
 
 

Re: Q19 - one theory to explain the sudden extinction

by ManhattanPrepLSAT1 Fri Dec 31, 1999 8:00 pm

Answer choice (A) undermines the causal conclusion by providing an example of the presumed effect without the presumed cause.

The argument concludes that toxic poisoning from angiosperms caused the sudden extinction of all dinosaurs, which led to the contorted positions we often see when we find dinosaurs.

To undermine a causal conclusion like this, stick to the script. The correct answer will (under normal circumstances) accomplish one of the following:

1. present an alternative cause
2. present an example of the presumed cause without the presumed effect
3. present an example of the presumed effect without the presumed cause

Any of the preceding will suffice, but choose wisely, since sometimes they make you decide amongst these options. If so, choose the answer choice that presents an alternative cause.

On a first run through of the answers, (A) and (E) stand out as the best candidates. Answer choice (A) presents option 3. Answer choice (E) presents option 2. So how do we decide? What's the difference between these answer choices? Well answer choice (A) mentions something that answer choice (E) does not - contorted positions. Remember the argument goes out of it's way to mention that the theory draws it's strongest support from the fact that it would help explain the contorted positions of many dinosaur fossils. Since answer choice (A) addresses the area where the theory draws it's strongest support it does more to undermine the conclusion.

Also, this argument is confusing because it's playing with more than one causal explanation. The toxins caused the extinction, and the toxins caused the contorted positions. It's one causal chain, but you do have to be careful.

(B) is irrelevant to whether toxins from angiosperms caused the contorted positions of dinosaur fossils.
(C) strengthens the conclusion, by providing a means for dinosaurs who were not eating angiosperms to also suffer the same fate.
(D) is irrelevant. The argument does not rely on an assumption that every toxic plant is produces amino-acid-based alkaloids nor does it assume that the only toxic plants were angiosperms.
(E) is consistent with the argument. And fails to address the theory's strongest support - contorted positions.

Does that answer your question?


#officialexplanation
 
clarafok
Thanks Received: 5
Forum Guests
 
Posts: 98
Joined: December 27th, 2010
 
 
trophy
Most Thankful
trophy
First Responder
 

Q19 - One theory to explain

by clarafok Sat Jan 22, 2011 10:17 pm

hello,

is A the answer because it weakens the argument by stating that many fossils of large mammals are also found in contorted positions, meaning it's not just dinosaurs but carnivores or mammals found today?

thanks!
User avatar
 
bbirdwell
Thanks Received: 864
Atticus Finch
Atticus Finch
 
Posts: 803
Joined: April 16th, 2009
 
 
 

Re: Q19 - One theory to explain

by bbirdwell Tue Jan 25, 2011 3:23 am

I think you're on the right track, though I'm not sure exactly what you mean by "carnivores or mammals found today." Just mammals is fine, and they don't even have to be found today.

The argument says "Dinos died by eating these plants. Mammals do not die by eating these plants. Also, dead dinos are found in contorted positions."

If I come along and say "lots of dead mammals are found in contorted positions, too," then, suddenly, dying in a contorted position is not good evidence for being poisoned by the plants, and a key piece of evidence has been shot out from under the conclusion.

Is that what you were thinking?
I host free online workshop/Q&A sessions called Zen and the Art of LSAT. You can find upcoming dates here: http://www.manhattanlsat.com/zen-and-the-art.cfm
User avatar
 
tamwaiman
Thanks Received: 26
Forum Guests
 
Posts: 142
Joined: April 21st, 2010
 
 
trophy
Most Thankful
 

Re: Q19 - one theory to explain the sudden extinction

by tamwaiman Sat Jun 04, 2011 4:24 am

Is (A) a good answer?

Since MANY is somewhat vague(maybe 10, or 10000+), we don't know how many is enough to weaken the argument. Moreover, even if there are many contorted mammal fossils, we cannot conclude that the "drug overdoses" theory is untenable. Finally, dying in contorted position could be caused by other reasons except for poisoned by Angiosperms, isn't (E) better from this point of view?

Thanks.
 
zainrizvi
Thanks Received: 16
Atticus Finch
Atticus Finch
 
Posts: 171
Joined: July 19th, 2011
 
 
trophy
First Responder
 

Re: Q19 - one theory to explain the sudden extinction

by zainrizvi Wed Oct 26, 2011 1:07 pm

I don't understand how option (E) is showing a cause without a effect. I thought it was just attacking the premise..
 
irene122
Thanks Received: 0
Forum Guests
 
Posts: 34
Joined: August 30th, 2011
 
 
 

Re: Q19 - one theory to explain the sudden extinction

by irene122 Wed Nov 09, 2011 8:07 pm

I think E is incorrect because it states "sometimes", the stimulus mentions "most" plant-eating dinosaur avoid angio--so the scope does not conflict therefore E does not attack the stimulus.

I hesitated when seeing "many" in A, because I recall that "many" is not sufficient to weaken causality. A weakens the stimulus by suggesting there's other poisoning and lethal plant/subjects could be eaten by large mammal--implying a possibility of alternative cause.

But on the other hand, since the stimulus states "most" plant-eating dinosaur avoid angio, the "many" fossil of large mammals in A could be the rest few that could eat angio and die in contorted postions. By this perspective, "many" in A seems insufficient to weaken the causality in the conclusion.

I am confused, could anyone please explain A and what A imples? Thanks a lot!
 
denis468
Thanks Received: 0
Forum Guests
 
Posts: 11
Joined: November 12th, 2012
 
 
 

Re: Q19 - One theory to explain

by denis468 Mon Nov 12, 2012 5:54 pm

19. One theory pointing to the extinction of dinasaurs point to a possibility of poisoning. Poison A appeared around the time of the extinction. Mammals that eat plants avoid those plants that contain poison because such plants usually have bitter taste. In addition, some mammals have livers that could neutralize the poison. However, were not capable of either detecting taste or neutralize such poisons. In addition, dinosaurs were found dead in contorted positions.
Check: What could undermine the argument? If mammals were found dead in contorted positions; mammals have been able to detect poisons and/or neutralize poisons. However, they where found dead in contorted positions. What does it tell us about the cause of their deaths? It suggests that there is something in common between deaths of both mammals and dinos, but it is not the poison present in plants that they were feeding on.
 
rbkfrye
Thanks Received: 0
Forum Guests
 
Posts: 10
Joined: February 22nd, 2013
 
 
 

Re: Q19 - One theory to explain

by rbkfrye Mon May 27, 2013 4:39 am

I agree with Irene, that's wack if (A) is the answer. "Many" should not be enough to break the causal chain:
"”as she pointed out, "many" is 100% compatible with what the prompt says, as it is not a relative numerical indicator ("many" says nothing about the percentage of mammals found this way)
"”the author practically trips over himself to make both statements about mammals surviving around angiosperms non-comprehensive ("most" plant-eaters; mammal livers "help" detoxify, but don't necessarily succeed).

I understood A to be a probable (since most *can* mean all) secondary conclusion before I even saw it on the answer list. If I had a dollar for every time other answers are marked wrong because of reading too much into adjectives like "many"... this is serious LSAC inconsistency as far as I'm concerned.
 
T.J.
Thanks Received: 0
Elle Woods
Elle Woods
 
Posts: 63
Joined: May 21st, 2013
 
 
 

Re: Q19 - One theory to explain

by T.J. Wed Jan 15, 2014 6:06 pm

I will talk about A and E on two fronts: argument core and whether "many" is sufficient to undermine the causal chain.

1. Argument core: the theory in this argument seeks its support from the evidence that many dinosaurs died in an ugly way. Thus, to undermine the theory, we alienate the premise from the conclusion, which is to say that not only dinosaurs died like that, many large mammals did that too. As mammals are biologically different from dinosaurs in terms of their detoxification abilities, the evidence from fossils fails to support the theory. In this aspect, A is superior to E, as it addresses the argument core.

2. The issue of many: from the posts that I've read above, some folks suspect that "many" is not sufficient to weaken the theory. I say yes it is enough to undermine the theory for the following reasons:
First, according to the stimulus "MANY dinosaur fossils are found...", the argument itself uses MANY to gain support. Hence, many is enough for weakening.
Second, if somebody says A causes B, but I got a piece of evidence suggesting that I got B without A. Then it is a fair game to say the causation does not really hold water. In this case, I don't even need MANY; one instance is enough to refute the causal chain (the presumed effect without the presumed cause as Matt puts it).
 
kjsmit02
Thanks Received: 2
Vinny Gambini
Vinny Gambini
 
Posts: 16
Joined: January 07th, 2015
 
 
 

Re: Q19 - one theory to explain the sudden extinction

by kjsmit02 Wed May 06, 2015 9:52 pm

mattsherman Wrote:Answer choice (A) undermines the causal conclusion by providing an example of the presumed effect without the presumed cause.

The argument concludes that toxic poisoning from angiosperms caused the sudden extinction of all dinosaurs, which led to the contorted positions we often see when we find dinosaurs.

To undermine a causal conclusion like this, stick to the script. The correct answer will (under normal circumstances) accomplish one of the following:

1. present an alternative cause
2. present an example of the presumed cause without the presumed effect
3. present an example of the presumed effect without the presumed cause

Any of the preceding will suffice, but choose wisely, since sometimes they make you decide amongst these options. If so, choose the answer choice that presents an alternative cause.

On a first run through of the answers, (A) and (E) stand out as the best candidates. Answer choice (A) presents option 3. Answer choice (E) presents option 2. So how do we decide? What's the difference between these answer choices? Well answer choice (A) mentions something that answer choice (E) does not - contorted positions. Remember the argument goes out of it's way to mention that the theory draws it's strongest support from the fact that it would help explain the contorted positions of many dinosaur fossils. Since answer choice (A) addresses the area where the theory draws it's strongest support it does more to undermine the conclusion.

Also, this argument is confusing because it's playing with more than one causal explanation. The toxins caused the extinction, and the toxins caused the contorted positions. It's one causal chain, but you do have to be careful.

(B) is irrelevant to whether toxins from angiosperms caused the contorted positions of dinosaur fossils.
(C) strengthens the conclusion, by providing a means for dinosaurs who were not eating angiosperms to also suffer the same fate.
(D) is irrelevant. The argument does not rely on an assumption that every toxic plant is produces amino-acid-based alkaloids nor does it assume that the only toxic plants were angiosperms.
(E) is consistent with the argument. And fails to address the theory's strongest support - contorted positions.

Does that answer your question?


Probably beating a dead horse at this point, as this question hasn't been addressed in awhile, but your response made it click with me. Thank you very much! (E) really doesn't weaken the argument at all, and is in fact completely consistent with the "MOST plant eating animals...." and the "mammals have livers that HELP detoxify..." language that's found in the prompt. It only makes sense that some of these mammals die from drug overdoses seeing this weak language in the supporting premises. Do we know if they die in contorted positions from this though? No. And these deaths in contorted positions are the main reasoning for dinos believing to have died from drug overdoses. However, (A) damages this reasoning. If MANY (weak language, but still substantial enough to be classified as 'many' rather than 'some') mammals are found in these same unusual positions, and there is an underlying assumption with support that mammals have biological ways to withstand these poisons, then how do these mammals still end up in contorted positions? Something besides drug overdoses via angiosperms are then likely the culprit if (A) is true.
 
Lsat 123
Thanks Received: 0
Vinny Gambini
Vinny Gambini
 
Posts: 4
Joined: July 30th, 2014
 
 
 

Re: Q19 - One theory to explain

by Lsat 123 Wed Jun 03, 2015 2:18 pm

am i wrong here or the answer choice doesn't undermine the theory itself, it only undermines the evidence brought to support the theory?????? pls help!
 
ganbayou
Thanks Received: 0
Atticus Finch
Atticus Finch
 
Posts: 213
Joined: June 13th, 2015
 
 
 

Re: Q19 - One theory to explain

by ganbayou Mon Jul 13, 2015 12:06 am

I was also not sure about the "many" in A.
But since this is trying to undermine the "cause"-"effect" relationship, maybe it is OK (even one example may destroy the relationship?)
At first I was not sure because in the stimulus, it says "Most" plant-eating mammals avoid these potentially lethal poisons, so I thought there is a possibility that some of them actually ate it and in that case A would not undermine the conclusion-maybe those ate it resulted in the form.
But then it says "mammals have livers that help detoxify such drugs" so even though some ate it, they cannot be poisoned...and this makes "without cause and still have the result" situation.
Still not sure about how strong "many" is in A, but even it is weak since this is the best among all, this is the answer I guess?
I'd appreciate it if anyone can confirm my thinking.

Thanks,
User avatar
 
rinagoldfield
Thanks Received: 308
Atticus Finch
Atticus Finch
 
Posts: 390
Joined: December 13th, 2011
 
 
 

Re: Q19 - One theory to explain

by rinagoldfield Thu Jul 16, 2015 1:40 pm

Thanks for your post, Ganbayou!

You are absolutely right that we don’t know the strength of “many.” It could refer to 3 mammals, 300, or 3,000.

(A)’s correct-ness reflects the nature of weaken questions. A weakener will almost certainly NOT break or disprove the argument. Rather, it will cast some kind of doubt on the argument. Mammals weren’t negatively impacted by angiosperms, so the author’s reliance on fossil contortion to prove that dinos DID eat angiosperms looks dubious in light of (A). (A) does not disprove the argument, but it is a little chink in the argument’s armor.
 
zdlsat
Thanks Received: 0
Vinny Gambini
Vinny Gambini
 
Posts: 13
Joined: July 23rd, 2014
 
 
 

Re: Q19 - One theory to explain

by zdlsat Tue Nov 17, 2015 3:20 am

rinagoldfield Wrote:Thanks for your post, Ganbayou!

You are absolutely right that we don’t know the strength of “many.” It could refer to 3 mammals, 300, or 3,000.

(A)’s correct-ness reflects the nature of weaken questions. A weakener will almost certainly NOT break or disprove the argument. Rather, it will cast some kind of doubt on the argument. Mammals weren’t negatively impacted by angiosperms, so the author’s reliance on fossil contortion to prove that dinos DID eat angiosperms looks dubious in light of (A). (A) does not disprove the argument, but it is a little chink in the argument’s armor.




Hi Rina,
I'm so confused about S/W question when argument is a causal relation. Many/some/several could S/W or not?

For this question, you said many is not strong enough, but it cast some doubt on the argument, so this is the correct answer.

But for this question Prep53 Section1 Q8
https://www.manhattanprep.com/lsat/foru ... -t817.html
The reason why E is incorrect is that several is not strong enough and it cannot indicate a correlation. However, Apply your principle here, Answer E casts some doubt, even though several is not strong enough. Why E can't be correct?

Applying above-mentioned principle to this question, Answer A is so weak. It doesn't indicate anything. Many mammals found in weird position, yet we can say many are not. Those in weird positions are probably just coincident. Answer A would not weaken a cause-effect argument.


Another example is that PT44S2Q15
Answer C also brings up a similar correlation situation as argument indicates, but this doesn't strengthen either.

I think the principle when confronting some/many answers in a S/W question, regarding causal relation, is really malleable.

In this question, PT52S3Q19, A is right because there's no other better answer. In PT53S1Q8, E is wrong because there's a better answer attacking assumption. In PT44S2Q15, answer C is wrong because there's a better answer fill the gap.

I'm really confused about these answers, which brings some/many other cases indicating correlation/cause, when in S/W questions, whether they are correct or not. It seems that it really depends on other answers.

Would you mind clarify some of my confusion?


Thank you so much!!!
User avatar
 
maryadkins
Thanks Received: 640
Atticus Finch
Atticus Finch
 
Posts: 1261
Joined: March 23rd, 2011
 
 
 

Re: Q19 - One theory to explain

by maryadkins Mon Nov 23, 2015 10:16 am

zdlsat Wrote:I think the principle when confronting some/many answers in a S/W question, regarding causal relation, is really malleable...It seems that it really depends on other answers.


YES. YES YES YES.

It is not a magic potion. It is a tool you can use in evaluating answer choices. Answer choices to Strengthen and Weaken questions do depend on the other answer choices. You are choosing the answer that most s/w.

Nice job reaching your own lesson here!

And if you're still confused, consult Matt's initial explanation above, again. It's EXCELLENT and very thorough.